If $4,780 is deposited in an account that pays 1.25% interest compounded annually, how much interest is in the account at the end of 8 years? A $5,279.44 B $500.44 C$ 478.00 D $499.44

Answers

Answer 1

We can calculate the interest as the difference between the future and the present value of the investment:

[tex]I=FV-PV[/tex]

The present value is $4780.

The annual interest rate is r=1.25/100=0.0125.

The number of years is 8, so n=8.

We can calculate the future value as:

[tex]\begin{gathered} FV=PV(1+r)^n \\ FV=4780\cdot(1+0.0125)^8 \\ FV=4780\cdot1.0125^8 \\ FV\approx4780\cdot1.1045 \\ FV\approx5279.44 \end{gathered}[/tex]

Then, we can calculate the interest as:

[tex]I=FV-PV=5279.44-4780=499.44[/tex]

Answer: D. $499.44


Related Questions

Is (2,7) a solution to the system of equations: y= x + 5 y= 4x + 1 Part 1 (circle 1) Yes No Part 2 Shows a proof of your answer

Answers

Answer

(2, 7) is not a solution to this system of equations.

The ordered pair doesn't fit into the second equation

Explanation

To check if the given ordered pair is a solution to this, we will insert the values into the two equations and if they fit into rach equation, then, the ordered pair is a solution for this.

y = x + 5

y = 4x + 1

(2, 7) means x = 2, y = 7

y = x + 5

7 = 2 + 5

7 = 7

y = 4x + 1

7 = 4(2) + 1

7 = 8 + 1

7 ≠ 9

The ordered pair doesn't fit into the second equation, hence, (2, 7) is not a solution to this system of equations.

Hope this Helps!!!

A regular hexagon has perimeter 72m. Find the area

Answers

A regular hexagon has six equal sides:

If you notice, a regular hexagon has exactly six equilateral triangles inside of it.

So, to find the area we could find the area of a triangle and then multiply this result by 6. This seems to be a little bit complicated to do, so, there's a formula to find the area of a regular hexagon, which is:

[tex]\begin{gathered} A=\frac{3\sqrt{3}s^2}{2} \\ Where: \\ s:Measure\text{ of a side of the regular hexagon} \end{gathered}[/tex]

We know that the measure of any side of our regular hexagon equals 12m, so, we could just replace in the equation:

[tex]A=\frac{3\sqrt{3}(12m)^2}{2}\rightarrow A=\frac{3\sqrt{3}(144m^2)}{2}\rightarrow A=216\sqrt{3}m^2[/tex]

Simplified, that's about 374.1 m2.

in a particular hospital, newborn babies were delivered yesterday. here are their weights (in ounces). 121 ,101 ,97 121,124 ,112 assuming that these weights constitute an entire population, find the standard deviation of the population. round your answer to two decimal places.

Answers

The standard population formula is:

[tex]\sigma=\sqrt[]{\frac{\sum ^{}_{}(x_{}-\mu)^2}{n}}[/tex]

where

x is the data points

μ is the mean of the data

and n is the number of data points

The mean is computed as follows:

[tex]\mu=\frac{\Sigma x}{n}[/tex]

In this case, the mean is:

[tex]\mu=\frac{121+101+97+121+124+112}{6}=\frac{676}{6}=112.67[/tex]

Then, the standard deviation of the population is:

[tex]\begin{gathered} \sigma=\sqrt[]{\frac{(121-112.67)^2+(101-112.67)^2+(97-112.67)^2+(121-112.67)^2+(124-112.67)^2+(112-112.67)^2}{6}} \\ \sigma=\sqrt[]{\frac{69.39+136.19+245.55+69.39+128.37+0.045}{6}} \\ \sigma=\sqrt[]{108.22} \\ \sigma=10.4 \end{gathered}[/tex]

Flying against the wind, an airplane travels 2010 kilometers in 3 hours. Flying with the wind, the same plane travels 10,530 kilometers in 9 hours. What is the rate of the plane in still air and what is the rate of the wind?

Answers

it is given that,

the distance travel against wind = 2010

time = 3 hrs

so, the relative speed is = distance/ time = 2010/3 = 670 km/hr

the distance travel with wind is 10,530

time 9 hrs

so,the relative speed is,= 10 530/9 = 1170 km/hr

let a = speed of plane ,

b = speed of wind

so, when plane travel with wind the the relative speed is ,

a + b = 1170

when travel against wind then the relative speed,

a - b = 670

sum the equation

a + b + a - b = 1170 + 670

2a = 1840

a = 920

put a = 920 in equation a + b = 1170

920 + b = 1170

b = 1170 - 920

b = 250

thus, the rate of plane is 920

rate of wind is 250

What is the determinant of H= 0 2 3-1 3 56 3 -2

Answers

[tex]-7[/tex]

1) Since this is a Matrix 3x3 we can make use of the Sarrus Rule to find the determinant of this Matrix:

[tex]\begin{bmatrix}0 & 2 & 3 \\ -1 & 3 & 5 \\ 6 & 3 & -2 \\ \end{bmatrix}[/tex]

2) We can do that by copying two columns to the right of the Matrix, and multiplying the entries, this way:

Now, let's add algebraically each diagonal and subtract from the other like this:

[tex]\begin{gathered} \det (H)=\lbrack60-9+0\rbrack-\lbrack54+4+0\rbrack \\ \det H)=51-58 \\ \det (H)=-7 \end{gathered}[/tex]

The sum of two numbers is30. The sum of4 times the larger and6 times the smaller is128. Find the numbers.

Answers

let

the smaller number = x

the larger number = y

x + y = 30

4y + 6x = 128

[tex]\begin{gathered} x+y=30 \\ 4y+6x=128 \\ x=30-y \\ 4y+6(30-y)=128 \\ 4y+180-6y=128 \\ -2y=128-180 \\ -2y=-52 \\ y=\frac{52}{2} \\ y=26 \\ x+y=30 \\ x+26=30 \\ x=30-26 \\ x=4 \end{gathered}[/tex]

The numbers are 4 and 26.

GEOMETRY: Express the volume of each cube below as a monomialNeeded fast!

Answers

Remember that

The volume of a cube is equal to

[tex]V=b^3[/tex]

where b is the long side of the cube

so

Part 19

we have that

[tex]b=7c^6d^2[/tex]

substitute in the formula

[tex]V=(7c^6d^2)^3[/tex]

Applying property of exponents

[tex]\begin{gathered} V=(7^3)(c^{(6\cdot3)})(d^{(2\cdot3)}) \\ V=342c^{(18)}d^6 \end{gathered}[/tex]

Part 20

we have

[tex]b=6r^7s^8[/tex]

substitute in the formula

[tex]\begin{gathered} V=(6r^7s^8)^3 \\ V=216r^{(21)}s^{(24)} \end{gathered}[/tex]

find the slope of the line

Answers

Solution

We can use the following points from the graph:

(0,-2) and (2,1)

And we can find the slope on this way:

[tex]m=\frac{y_2-y_1}{x_2-x_1}=\frac{1+2}{2-0}=\frac{3}{2}[/tex]

Then we can find the intercept on this way:

1= 3/2 (2)+b

b= 1-3= -2

Then the equation would be:

y= 3/2 x -2

How do I know if 6.209 is greater or lesser than 6.29

Answers

Since,

[tex]6.29-6.209=0.081[/tex]

The difference gives us a positive value.

So, 6.29 is grater than 6.209

Answer: Add a 0 to 6.29 and compare the numbers to see which one is greater

6.29  (or 6.290) is greater than 6.209 since the hundrenth in 6.29 is 9 and the other hundrenth for 6.209 is 0.

A landscaper's truck is filled with ton of gravel.The gravel is shared equally among 3 projects.3. Write and solve a division equation to find how muchgravel each project will get. Explain your reasoning.

Answers

Step 1

Let n represent the weight of tons of gravel.

Step 2

The gravel is shared equally among the 3 projects.

Step 3

Write the division equation

Each project will get

[tex]=\text{ }\frac{n}{3}\text{ tons of gravel}[/tex]

What is the value of X?

Answers

Check the picture below.

Make sure your calculator is in Degree mode.

Geometry- Need help` brainly logged me out w my other tutor who explained it so if u see this miss tutor my bad

Answers

Explanation

"Reason" means a mathematical justification for the assert on the left. "Given" means something that doesn't need justification; it's an assumption.

The first statement,

[tex]\bar{FG}\cong\bar{FJ},[/tex]

is given.

The second reason is Base Angles Theorem. Note the word angles in the middle. Its corresponding statement on the left must involve angles. There is only one option involving angles:

[tex]\measuredangle G\cong\measuredangle J.[/tex]

Finally, statement 3 is also in the assumptions made above (tagged by Given:). It's also Given.

Answer

The reason #1 is Given.

The statement 2 is

[tex]\measuredangle G\cong\measuredangle J.[/tex]

The reason #3 is Given.

Jim stocks shelves at a grocery store. He aerns $8.60 per hour for 37.5 hours each week. One week a large shipment arrives late and Jim is asked to work overtime at 1.5 times his regular rate. He works 4.5 hours for overtime. What are his total earnings for the week?

Answers

Explanation

Since Jim earns 8.60 per hour for 37.5 hours each week. His normal earnings for a week becomes,

[tex]earnings=8.60\times37.5=322.5\text{ dollars}[/tex]

During the late shipment period, Jim had to earned an overtime payment at 1.5 times his regular rate. This means per overtime hour he would earn

[tex]1.5\times8.60=12.9[/tex]

Therefore, for 4.5 hours for overtime, we will have;

[tex]12.9\times4.5=58.05\text{ dollars}[/tex]

Hence, altogether, he would make;

[tex]322.5dollars+58.05dollars=380.55\text{ dollars}[/tex]

Answer: 380.55 dollars

I have been stuck on this for a while, your help would be most appreciated!

Answers

Answer:I Don't Know What She Said

Step-by-step explanation:

Consider a triangle ABC like the one below. Suppose that A = 35°, C = 68°, b = 32. (The figure is not drawn to scale.) Solve the triangle. Round your answers to the nearest tenth.

Answers

Solution.

Given the triangle

[tex]\begin{gathered}

Using sine rule,

[tex]\frac{a}{sinA}=\frac{b}{sinB}=\frac{c}{sinC}[/tex][tex]\begin{gathered} \frac{a}{sin35}=\frac{32}{sin77} \\ a=\frac{32\text{ x sin35}}{sin77} \\ a=18.84 \\ a=18.8(nearest\text{ tenth\rparen} \end{gathered}[/tex][tex]\begin{gathered} \frac{a}{sinA}=\frac{c}{sinC} \\ \frac{18.84}{sin35}=\frac{c}{sin68} \\ \end{gathered}[/tex][tex]\begin{gathered} c=\frac{18.84\text{ x sin68}}{sin35} \\ c=30.45 \\ c=30.5(nearest\text{ tenth\rparen} \end{gathered}[/tex]

(MP Reason An internet company spends half their yearly profits on advertising for the next year. Of the remaining half, they spend 1/5 on new computers. What fraction of the total profits does the company spend on new computers? Use the number line to show how you can to make his bowl? Write an equati Harcourt Pub 4 find the fraction. 1 o Module 8. Lesson 3

Answers

Let the total profit be represented by 1

The internet company spends half their yearly profits on advertising for the next year. The amount of the profit spent on advertising is 1/2 = 0.5

The amount left is 1 - 0.5 = 0.5

Of the remaining half, they spend 1/5 on new computers. It means that the amount spent on new computers is

1/5 * 0.5 = 0.1 = 1/10

Therefore, the fraction of the total profits does the company spend on new computers is

0.1/1 = 0.1 = 1/10

The number line is shown below

The solutions to a quadratic equation are -2 and 6. What is the equation of its axisof symmetry?

Answers

General form of a quadratic equation is

[tex]ax^2+bx+c=0[/tex]

another form is

[tex](x+h)(x+k_{})=0[/tex]

where h and k are the number opposite by the sign of the solutions, then on this case the values of h and k are 2 and -6

[tex](x+2)(x-6)=0[/tex]

Our equatio is a parabola then if we find the vertex we are finding the axis of simmetry

to find the vertex we trasnforme ou equation to the general form of a quadratic equation multipliying parenthesis

[tex]\begin{gathered} (x\times x)+(x\times-6)+(2\times x)+(2\times-6)=0 \\ x^2-6x+2x-12=0 \\ x^2-4x-12=0 \end{gathered}[/tex]

now take the equation and derivate

[tex]\begin{gathered} 2x-4-0=0 \\ 2x-4=0 \end{gathered}[/tex]

if we solve x we find the coordinate x of the vertex and the axis of simmetry

then

[tex]\begin{gathered} 2x-4=0 \\ 2x=4 \\ x=\frac{4}{2} \\ \\ x=2 \end{gathered}[/tex]

axis of Symmetry is x=2

1 8. Matt wants to purchase a gasoline motor scooter. The gas mileage is 75 miles for each gallon of gasoline. How many miles will Matt be able drive on 5 gallons of gasoline? 2

Answers

Answer:

375 miles

Explanation:

The gas mileage of the scooter = 75 miles for each gallon of gasoline.

Therefore:

If 1 gallon will cover a distance of 75 miles

Then: 5 gallons will cover a distance of:

5 x 75 miles

=375 miles

Matt will be able to drive 375 miles on 5 gallons of gasoline.

Three cards are drawn from an ordinary deck and not replaced find the probability of the following P(getting 3 queens)P(getting an ace and king and queen)P(getting club and spade and heart)P(getting 3 diamonds)Answer the following problems using multiplication rule make sure to reduce your fraction

Answers

a) P(3 queens) ) = 4/52*3/51*2/50 = 1/5525

b) P(ace,king, queen) = 4/52*4/51*4/50 = 8/16575

c) P( ) = 10/52 * 10/51*10/50 = 5/663

d) P(3 diamonds) = 10/52 * 9/51* 8/50 = 6/1105

Really need help with 11 and 12 just started learning this today and I don't quite understand it would really appreciate it

Answers

11.

Given:

[tex]y=x^2[/tex]

Differentiate with respect to x, we get

[tex]\frac{dy}{dx}=2x[/tex]

The given point is (3,9)

Substitute x=3 in the derivative, we get

[tex]\frac{dy}{dx}=2\times\times3=6[/tex]

Hence the slope is 3.

12.

Given:

[tex]y=x^2+4[/tex]

Differentiate with respect to x, we get

[tex]\frac{dy}{dx}=2x+0[/tex]

The given point is (0,4)

Substitute x=0 in the derivative, we get

[tex]\frac{dy}{dx}=2(0)+0=0[/tex]

Hence the slope is 0.

Question: What is the solution to this system of linear equations? 8x + 2y = 2 and x + 3y 2 = 14 I NEED The claim, evidence and reasoning. Please I need help

Answers

[tex]\begin{gathered} 8x+2y=2\ldots\ldots.(1) \\ x+3y=14 \\ x=14-3y\ldots\text{.}(2) \\ \text{Substitute the value of x in equation (1).} \\ 8(14-3y)+2y=2 \\ 112-24y+2y=2 \\ 112-2=22y \\ y=5 \\ Now,\text{ from equation (2)} \\ x=14-3y \\ x=14-3\times5 \\ x=14-15 \\ x=-1 \\ \text{So, the solution is(-1,5).} \end{gathered}[/tex]

help my brother the other one I tried was a scam

Answers

1. Given the integers a and b, a common multiple is a positive integer that is divisible by both a and b.

2. The Least Common Multiple (LCM ) is the smallest positive integer that is divisible by both a and b.

3. Multiples of 6: 6 (= 6x1), 12 (= 6x2), 18 (= 6x3), 24 (= 6x4), ...

Multiples of 4: 4 (= 4x1), 8 (= 4x2), 12 (= 4x3), 16 (= 4x4), ...

The smallest number of these two lists is 12, then the LCM between 6 and 4 is 12.

At the start of a research study, a colony of penguins had a population of 20,000. One year later, it had a population of 21,200.Assuming the population of the colony has grown exponentially, which expression best models thepopulation? Let t represent the time in years from the start of the research study.1,200(1.015)^t20,000 (1.06)^4t21,200 (1.012)^t20,000 (1.06)^tAssuming the colony continues to grow at the same rate, what will the population of the colony be 4 years after the start of the research study?Round your answer to the nearest whole number.

Answers

Solution:

An exponential function is generally expressed as

[tex]\begin{gathered} y=a(b)^t\text{ ----- equation 1} \\ \end{gathered}[/tex]

Given that in a research study, a colony of penguins had a population of 20,000.

This implies that

[tex]\begin{gathered} when\text{ t=0,} \\ 20,000=ab^0 \\ \Rightarrow20000=a\times1\text{ \lparen where b}^0=1) \\ thus, \\ a=20000 \end{gathered}[/tex]

Substitute the value of a into equation 1.

Thus,

[tex]y=20000(b)^t\text{ ----- equation 2}[/tex]

One year later, it had a population of 21,200. This implies that when t equals 1, we substitute the values of 21200 and 1 for y and t respectively into equation 2.

This gives

[tex]\begin{gathered} 21200=20000(b)^1 \\ \Rightarrow21200=20000b \\ divide\text{ both sides by the coefficient of b, which is b.} \\ thus, \\ \frac{21200}{20000}=\frac{20000b}{20000} \\ \Rightarrow b=1.06 \end{gathered}[/tex]

Substitute the obtained value of b into equation 2.

Thus, the expression that best models the population is

[tex]20,000(1.06)^t[/tex]

Assuming the colony grows at the same rate, the population of the colony after 4 years is evaluated by solving for y when the value of t is 4.

Thus,

[tex]\begin{gathered} y=20,000(1.06)^t \\ when\text{ t=4, we have} \\ y=20,000(1.06)^4 \\ =20000\times(1.06)^4 \\ =20000\times1.26247696 \\ \Rightarrow y=25249.5392 \\ \therefore y=25250\text{ \lparen nearest whole number\rparen} \end{gathered}[/tex]

Hence, after 4 years the population of the colony will be 25250 penguins (nearest whole number).

What type of number is 3 - 77iChoose all answers that apply:A. RealB. ImaginaryC. Complex

Answers

[tex]\begin{gathered} \Rightarrow3-77i \\ When\text{ the number is having i, then it is treated as the imaginary number.} \end{gathered}[/tex]

True or False? In a two column proof, the right column contains a series of deductions. (Geometry)

Answers

In a two column proof the left column contains a series of statements. The reasons why these statements are true are given in the right column. This reasons are deduction made from the data provided by the problem. Then the statement "In a two column proof, the right column contains a series of deductions." is True.

An experiment consists of drawing 1 card from a standard 52-card deck. What is the probability of drawing a queen ? Question content area bottomPart 1The probability of drawing a queen is   enter your response here .(Type an integer or a simplified fraction.)

Answers

Given:

Total number of cards = 52

Number of cards drawn = 1

Then:

Number of ways of drawing a card from 52 cards

[tex]\begin{gathered} =^{52}C_1 \\ =52 \end{gathered}[/tex]

Number of queens in a deck of cards = 4

Number of ways that one card is a queen

[tex]\begin{gathered} =^4C_1 \\ =4 \end{gathered}[/tex]

Probability of drawing a queen

[tex]\begin{gathered} =\frac{\text{ Number of favorable cases}}{\text{ Total number of cases}} \\ =\frac{4}{52} \\ =\frac{1}{13} \end{gathered}[/tex]

Final answer: 1/13

what is the answer to number 9 and how do i solve?

Answers

Given:

[tex]T=\frac{1}{r}\frac{N_{\infty}-N_0}{N_0}[/tex]

From question (1)

[tex]r=2[/tex]

and given:

[tex]\begin{gathered} N_{\infty}=3.3 \\ N_0=10 \end{gathered}[/tex]

we will find the daily cases beak as follows:

[tex]undefined[/tex]

The Senior classes at High School A and High School B planned separate trips to the indoor climbing gym. The senior class at High school A Rented and Filled 6 vans and 7 buses with 471 students. High School B rented and Filled 5 vans and 9 buses with 573 students. Every van had the same number of students in it as did the buses. How many students can a van carry? How many Students can a bus carry?

Answers

Answer: A van can carry 12 students

A bus can carry 57 students

Explanation:

Let x represent the number of students that a van can carry

Let y represent the number of students that a bus can carry.

The senior class at High school A Rented and Filled 6 vans and 7 buses. This means that

the number of senior class students that the van carried is 6 * x = 6x

the number of senior class students that the bus carried is 7 * y = 7y

If both vehicles were filled with 471 students, it means that

6x + 7y = 471 equation 1

High School B rented and Filled 5 vans and 9 buses. This means that

the number of High School B students that the van carried is 5 * x = 5x

the number of High School B students that the bus carried is 9 * y = 9y

If both vehicles were filled with 573 students, it means that

5x + 9y = 573 equation 2

We would solve both equations by applying the method of elimination. To eliminate x, we would multiply equation 1 by 5 and equation 2 by 6. The new equations are

30x + 35y = 2355 equation 3

30x + 54y = 3438 equation 4

Subtracting equation 3 from equation 4, we have

30x - 30x + 54y - 35y = 3438 - 2355

19y = 1083

y = 1083/19

y = 57

Substituting y = 57 into x equation 1, we have

6x + 7 * 57 = 471

6x + 399 = 471

6x = 471 - 399 = 72

x = 72/6

x = 12

Thus,

A van can carry 12 students

A bus can carry 57 students

If f(x)=x²-20 and g(x) = 4+3x, then f(g(-3)) =

Answers

The value of function f(g(-3)= 5.

What is composite function?

A composite function is generally a function that is written inside another function. Composition of a function is done by substituting one function into another function. For example, f [g (x)] is the composite function of f (x) and g (x).

Given, f(x)=x²-20 and g(x) = 4+3x

first we will find

f(g(x)) = f(4+3x)^2-20

=9x^2+16+24x-20

=9x^2+24x-4

Now to find, f(g(-3)), substitute x=-3,

= 9(9)+24(-3)-4

=81-72-4

=5

To know more about composite function, visit:

https://brainly.com/question/4075277

#SPJ1

Blair purchases the T-shirts from Company B. She needs to add a 75markup to her total cost to make a profit when she sells the shirts at thcarnival. How should Blair determine the selling price of each T-shirt athe carnival?Drag a number into each box to make the statements true.She canThe total cost Blair paid for one T-shirt is $calculate the markup by multiplying her total cost byBlair will sell each T-shirt at the carnival for $

Answers

[tex]\text{ markup \% = }\frac{\text{ selling price - cost}}{\text{ cost}}\cdot100[/tex]

Isolating selling price,

[tex]\begin{gathered} \frac{\text{ markup \% }}{100}\text{= }\frac{\text{ selling price - cost}}{\text{ cost}} \\ \frac{\text{ markup \% }}{100}\cdot\cos t=\text{selling price - cost} \\ \frac{\text{ markup \% }}{100}\cdot\cos t+cost=\text{selling price} \\ \cos t(\frac{\text{ markup \% }}{100}+1)=\text{ selling price} \end{gathered}[/tex]

The markup is 75 %. Supposing that the cost of a T-shirt is $6.00, then the selling price will be:

[tex]\begin{gathered} \text{selling price = 6.00}\cdot(\frac{75}{100}+1) \\ \text{selling price = 6.00}\cdot(0.75+1) \\ \text{selling price = 6.00}\cdot1.75 \\ \text{selling price = \$}10.5 \end{gathered}[/tex]

Other Questions
I need help answering this hi help I've been trying to solve this for an hour and this is due in 10 minutes I just really need the correct answer please help b. How many square meters does an 80. ft2 rug occupy? (1 ft = 12 in.) Here is a graph representing the relationship in question 3. Interpret the set of coordinates (30, 7).children's tickets sold60555045403025205(30, 7)10 15 20 25 30 35 40adult tickets soldO 7 children's tickets and 30 adult tickets were soldO 30 children's tickets and 7 adult tickets were soldO When 7 tickets are sold, $30 is collectedO (30, 7) is not a possible solution I. Ccile and her family are eating dinner. Complete their statements with the approprite definite, indefinite, or partitive article. 1. Il n'y a pas __ sel dans la soup.2. Tu veux ___ salade?3. Tu prends __ tarte comme dessert?4. Je n'aime pas beaucoup __ carottes5. Vous voulez ___ fraises? midterm 3 In an effort to stretch already thin educational budgets, a countys school cut home economics from their offered programs. Fast forward to 10 years down the road, which statement shows the MOST likely consequence of that choice? A. The county now has a generation of young adults who eat mostly fast food and take out. B. The county has a higher rate of college graduates and the median income has risen. C. The county has seen little to no change that can be traced back to the decision to cut the program. D. The county now has a generation of young adults who are opening new bars, breweries, and restaurants. wellness & nutrition Question 7b: Let g(x) be a polynomial function. Name all horizontal andvertical intercepts of the graphsg(x) = (x - 1)2 (x + 2)Horizontal intercepts: 1, -2; vertical intercepts: 2Horizontal intercepts: 2, vertical intercepts: 1,-2Horizontal intercepts: -1, 2, vertical intercepts: 4Horizontal intercepts: 4, vertical intercepts: -1,2 A survey of 85 persons was conducted at TCC, and it was found that 54 persons carried a cell phone, 19 persons carried a tablet computer, and 16 carried both a cell phone and a tablet.How many people carried a cell phone or a tablet?How many people carried neither a cell phone nor a tablet?How many people carried a cell phone only?How many people carried a tablet but not a cell phone? Find the area of each rectangle using the given information.(A=W x H)Question 9:6 and a height of 36 inches. Polls have found that inflation is consistently a top issue for voters across the country. What was the overall u. S. Inflation rate in september, compared with a year before?. Brian just completed a client needs assessment and his client confirmed that she will require profitability tracking, and needs to create budgets and purchase orders in her accounting solution. What is the minimum quickbooks online version suitable for brian's client?. a(n) ____________ involves interviewing business people at their offices concerning industrial products or services, a process that is very expensive. write an equation of a line passing through the point (-6,-3) and perpendicular to JK with J (-2, -7) and K (6,5) Square root of 4n^2+4n^2PLEASE HELPGiving brainliest b.InOut133171066co38Rule: how would I solve the system of equations using elimination?8x - 5y = 114x - 3y =5 Find the number that makes the ratio equivalent to 1:2 5:_ The partial ionic character of a bond is related to the difference in the electronegativities of the bonded elements. The greater the en, the the percent ionic character of the bond. Although there are many bonds that have 0% ionic character, no bond has 100% ionic character, which indicates that electron occurs to some extent in every bond. this graphic organizer is being used to classify triangles based on their angle measures or sideways which list shows all of the ways that strangle could be classified Chris tries to copy